Вы находитесь на странице: 1из 7

PHY3063 Spring 2006 R. D.

Field

PHY 3063 Exam 2 Solutions


Problem 1 (25 points): Consider an “atom” consisting of a proton and a muon. A proton has an
electric charge +e and rest mass energy Mpc2 ≈ 940 MeV. A muon is an elementary particle with
charge –e similar to an electron but with a mass that is 207 times the mass of an electron (mec2 ≈
0.511 MeV). Use Bohr’s model of an atom to calculate the following:
(a) (8 points) The radius of the first Bohr orbit (i.e. the ground state) of the muonic “atom” (in
fm).
Answer: 284 fm
Solution: Bohr’s model tell us that
1 me me 1 o o
r1 = r0 = r0 ≈ (0.529 A) ≈ 2.84 × 10 − 3 A = 284 fm ,
Z µ 186me 186
where I used
mM 207 me M p 1
µ≡ = = 207me ≈ 186me
m + M 207 me + M p 1 + 207me / M p
and
De o
r0 = ≈ 0.529 A .
α
(b) (8 points) The ground state energy of the muonic “atom” (in keV).
Answer: -2.53 keV
Solution: Bohr’s model tell us that
µ
E1 = Z 2 E0 = 186(−13.6eV ) = −2,530eV = −2.53keV ,
me
where
1 2 Ke 2 1
E0 = − α (mec ) = −13.6eV and α =
2
≈ .
2 hc 137
(c) (9 points) The ground state energy of the hydrogen atom can be estimated using the
uncertainty relation. For a hydrogen atom in the ground state the uncertainty ∆r in the position
of the electron is approximately the 1st Bohr radius, r0, consequently the elecrron’s momentum
pr is uncertain by about h / ∆r . Compute the ground state energy of the hydrogen atom (in eV),
pr2 Ke 2
E= − ,
2me r
by setting r = r0 and pr = ∆pr = h / r0 , where r0 is the Bohr radius of the ground state.
Answer: -13.6 eV
Solution: We see that
pr2 Ke 2 h2 Ke 2 h 2α 2 Ke 2α
E= − ≈ − = −
2me r 2me r02 r0 2me (h /(me c)) 2 h /(me c)
1 1
= α 2 (me c 2 ) − α 2 (me c 2 ) = − α 2 (me c 2 ) = −13.6eV
2 2

Exam 2 Solutions Page 1 of 7 March 9, 2006


PHY3063 Spring 2006 R. D. Field

Problem 2 (25 points): The parity operator, Pop, is defined by


Popψ ( x ) = ψ ( − x ) .

(a) (10 points) Prove that the parity operator is hermitian. Remember hermitian operators have
the property that O↑op = Oop where
∞ ∞

∫ [O ψ 2 ( x )] ψ 1 ( x ) dx = ∫ψ 2* ( x )Oop↑ ψ 1 ( x ) dx .
*
op
−∞ −∞
Solution: We see that
+∞ +∞

∫ [P ψ ]
( x ) ψ 1 ( x ) dx = ∫ψ 2∗ ( − x )ψ 1 ( x ) dx ,
*
op 2
−∞ −∞
but
+∞ −∞ +∞ +∞

∫ψ ( − x )ψ 1 ( x ) dx = − ∫ψ ( y )ψ 1 ( − y ) dy = ∫ψ ( y )ψ 1 ( − y ) dy = ∫ψ 2∗ ( x )ψ 1 ( − x ) dx ,
∗ ∗ ∗
2 2 2
−∞ +∞ −∞ −∞
where I changed variables with y = -x. Thus,
+∞ +∞ +∞ +∞

∫ [P ψ ]
2 ( x ) ψ 1 ( x ) dx = ∫ ψ 2 ( − x )ψ 1 ( x ) dx = ∫ ψ 2 ( x )ψ 1 ( − x ) dx = ∫ψ 2 ( x ) Popψ 1 ( x ) dx
* ∗ ∗ ∗
op
−∞ −∞ −∞ −∞
and hence P↑op = Pop which means that Pop is hermitian.
(b) (5 points) Prove that the eigenvalues of the parity operator are real.
Solution: Let Pop|ψ> = λ|ψ> where λ is a constant and |ψ> is the normalized eigenket (i.e.
<ψ|ψ> = 1). We see that
<ψ|Pop|ψ> = λ<ψ|ψ> = λ = (<ψ|P↑op|ψ> )*= (<ψ|Pop|ψ> )* = λ∗
and therefore λ is real since it is equal to its complex conjugate.
(c) (5 points) Prove that the eigenvalues of the parity operator are +1 and -1.
Solution: Let Pop|ψ> = λ|ψ> where λ is a real constant and |ψ> is the normalized eigenket. We
see that
<ψ|ψ> = <ψ|1|ψ> = <ψ |Pop Pop|ψ> = λ2<ψ|ψ>,
where I used Pop Pop = 1. Hence λ2 = 1, which means that λ = -1 or λ = +1.
(d) (5 points) Prove that if ψ(x) is an eigenstate of both Pop and the Hamiltonian operator Hop
then Pop and Hop commute (i.e. [Pop,Hop] = 0).
Solution: Let Hop|ψ> = h|ψ> and Pop|ψ> = λ|ψ>. We know that h and λ are real constants. We
see that
[Pop,Hop]|ψ>= (PopHop- HopPop)|ψ> = (λh- hλ)|ψ> = 0
and hence [Pop,Hop].

Exam 2 Solutions Page 2 of 7 March 9, 2006


PHY3063 Spring 2006 R. D. Field

Problem 3 (50 points): Consider an electron with mass me


confined within an infinite square well defined by Infinite Square Well

V(x) = 0 for 0 < x < L, V = +infinity V = +infinity


V(x) = +∞ otherwise.
(a) (5 points) Using Schrödinger’s equation calculate the allowed
stationary state eigenfunctions ψn(x), where the complete
wavefunctions are given by Ψn ( x, t ) = ψ n ( x )e n . Normalize
− iE t / h
0 L x

the eigenfunctions so that the probability of finding the electron somewhere in the box is one.
2
Answer: ψ n ( x) = sin( nπx / L)
L
Solution: For the region outside of 0 < x < L ψ ( x) = 0 and inside the region
h 2 d 2ψ ( x) d 2ψ ( x) h 2k 2
− = Eψ ( x ) or = − k 2
ψ ( x ) with E =
2me d 2 x d 2x 2me
The most general solution is of the form
ψ ( x) = A sin(kx) + B cos(kx) .
The boundary condition at x = 0 givesψ (0) = B = 0 and the boundary condition at x = L gives
ψ ( L) = A sin(kL) = 0 which implies that kL = nπ with n = 1, 2, 3,… Thus,
n 2π 2h 2
ψ n ( x) = A sin(nπx / L) with En = . The normalization is arrived at by requiring that
2mL2
+∞ L nπ nπ
LA2 LA2  y sin(2 y )  LA2
∫ψ n ( x)ψ n ( x)dx = 1 = A ∫ sin (nπx / L)dx = nπ ∫0
∗ 2 2
sin 2
( y ) dy =  −  =
−∞ 0
nπ  2 4 0 2
Thus, A = 2 / L . These states are called stationary because the probability density and all the
expectation values are independent of time.

(b) (5 points) Show that the wavefunctions Ψn(x,t) correspond to states with definite energy (i.e.
show that ∆E = 0).
Solution: We see that
+∞ +∞ +∞
∂Ψ ( x, t )
< E > n = ∫ Ψn∗ ( x, t ) H op Ψn ( x, t )dx = ih ∫ Ψn∗ ( x, t ) n dx = En ∫ Ψn∗ ( x, t )Ψn ( x, t )dx = En
−∞ −∞
∂t −∞
and
+∞ +∞ +∞
∂ 2 Ψn ( x, t )
< E > n = ∫ Ψn ( x, t ) H op Ψn ( x, t )dx = −h ∫ Ψn ( x, t )
2 ∗ 2 2 ∗
dx = En ∫ Ψn∗ ( x, t )Ψn ( x, t )dx = En2 .
2

−∞ −∞
∂t 2
−∞
Hence
( ∆E ) n = < E 2 > n − ( < E > n ) 2 = 0 .
(c) (5 points) Calculate the allowed energy levels, En, of the system. Express your answer in
terms of the Compton wavelength of the electron, D e = h /( me c ) , and the rest mass energy of the
electron, mec 2 . What is the ground state energy (in MeV) for the case L = D e ? (Note that mec2
= 0.511 MeV.)

Exam 2 Solutions Page 3 of 7 March 9, 2006


PHY3063 Spring 2006 R. D. Field

2
n 2π 2  D e  π2
Answer: En =   e ( m c 2
) , E1 = (me c 2 ) ≈ 2.52MeV
2  L 2
Solution: From part (a) we see that k = nπ/L and hence
2
h 2 k 2 n 2π 2h 2 n 2π 2  D e 
En = = = 2
  (me c ) .
2m 2mL2 2  L
The ground state corresponds to n = 1 and hence
2
π 2  De  π2 π2
E1 =   (me c ) L
2
→ (me c 2 ) =
=D e
(0.511MeV ) ≈ 2.52MeV .
2  L 2 2

(d) (5 points) Show that the states, ψ n (x) , form an orthonormal set. Namely, show that
+∞

∫ψ ( x)ψ n ( x)dx = δ mn .

m
−∞
Solution:
+∞ L π
2 2
∫ψ m ( x)ψ n ( x)dx =
∫ sin( mπx / L) sin( nπx / L)dx = ∫ sin( my ) sin( my )dy = δ mn

−∞
L0 π 0
th
(e) (5 points) Calculate <x> for n state stationary state.
L
Answer: < x > n =
2
Solution:
+∞ L 2 nπ
2 2 L 
< x >= ∫ψ ( x)( x) opψ n ( x)dx = ∫ x sin 2 (nπx / L)dx = 

 ∫ y sin
2
( y )dy
L  nπ 
n
−∞
L0 0

2 L  y 2 y sin( 2 y ) cos(2 y )  L
=  −
2 
−  =
(nπ )  4 4 8 0 2

(f) (5 points) Calculate <px> for nth state stationary state.


Answer: < px > n = 0
Solution:
+∞ +∞
 ∂ 
< p x >= ∫ψ ( x)( p x ) opψ n ( x)dx = ∫ψ n∗ ( x) − ih ψ n ( x)dx

n
−∞ −∞  ∂x 

2 nπ
L
2
= −i h
L L 0∫ sin( nπx / L) cos(nπx / L)dx = −ih ∫ sin( y ) cos( y )dy
L 0

2  sin 2 ( y ) 
= −ih   =0
L  2  0
Note that
d<x>
< px >= m .
dt

Exam 2 Solutions Page 4 of 7 March 9, 2006


PHY3063 Spring 2006 R. D. Field

(g) (10 points) Suppose the electron in this infinite square well has an initial wave function at t =
0 which is an equal mixture of the first two stationary states:
Ψ ( x,0) = A[ψ 1 ( x) + ψ 2 ( x)] .
What is the normalization A? If you measure the energy of this particle, what are the possible
values you might get, and what is the probability of getting each of them? What is the
expectation value of the energy for this state?
5π 2 h 2
Answer: A = 1 / 2 , E1 with probability ½ and energy E2 with probability ½, < E >=
4ma 2
Solution: The normalization is arrived at by requiring that
+∞ +∞
∗ 2 ∗ ∗
[ ]
∫ Ψ ( x,0)Ψ ( x,0)dx = 1 = A ∫ ψ 1 ( x) +ψ 2 ( x) [ψ 1 ( x) +ψ 2 ( x)]dx
−∞ −∞
+∞
( )
= A2 ∫ ψ 1∗ ( x)ψ 1 ( x) + ψ 1∗ ( x)ψ 2 ( x) + ψ 2∗ ( x)ψ 1 ( x) + ψ 2∗ ( x)ψ 2 ( x) dx = 2 A2
−∞

and A = 1 / 2 . Thus,
Ψ ( x, t ) = c1ψ 1 ( x)e − iE1t / h + c2ψ 2 ( x)e − iE 2 t / h ,
where c1 = c2 = 1 / 2 which means that you get energy E1 with probability ½ and energy E2 with
probability ½. The expectation value of E is
1  π 2h 2 4π 2h 2  5π 2h 2
< E >= 12 E1 + 12 E2 =  + =
2  2ma 2 2ma 2  4ma 2
(h) (10 points) Suppose as in part (d) the electron in this infinite square well has an initial wave
function at t = 0 which is an equal mixture of the first two stationary states:
Ψ ( x,0) = A[ψ 1 ( x) + ψ 2 ( x)] .
What is the probability density ρ ( x, t ) =| Ψ ( x, t ) |2 for this state. Does it depend on time? What is
<x> and <px> for this state. Do they depend on time?
L 32  8h ( E − E1 ) 3π 2h
Answer: < x >= 1 + 2 cos(∆ωt )  , < px >= − sin(∆ωt ) , where ∆ω = 2 =
2  9π  3L h 2mL2
Solution: From part (g) we have
1 1
Ψ ( x, t ) = ψ 1 ( x ) e − i ω1 t + ψ 2 ( x ) e − iω 2 t ,
2 2
where ωn = En / h . We see that
ρ ( x, t ) =| Ψ ( x, t ) |2 = 12 | ψ 1 ( x) |2 + 12 | ψ 1 ( x) |2 +ψ 1 ( x)ψ 2 ( x) Re(e −i (ω 2 −ω1 ) t
)
= 12 | ψ 1 ( x) |2 + 12 | ψ 2 ( x) |2 +ψ 1 ( x)ψ 2 ( x) cos(∆ωt )
where
( E2 − E1 ) 3π 2h
∆ω == .
h 2mL2
We see that ρ(x,t) depends on time. Also, we see that

Exam 2 Solutions Page 5 of 7 March 9, 2006


PHY3063 Spring 2006 R. D. Field

+∞ +∞ +∞
< x >= ∫ Ψ ( x, t )( x)op Ψ ( x, t )dx =
∗ 1
2 ∫ψ

1 ( x) xψ 1 ( x)dx + 1
2 ∫ψ

1 ( x) xψ 1 ( x)dx
−∞ −∞ −∞
+∞
+ cos(∆ωt ) ∫ xψ 1 ( x)ψ 2 ( x)dx
−∞
+∞
= < x >1 + < x > 2 + cos(∆ωt ) ∫ xψ 1 ( x)ψ 2 ( x)dx
1
2
1
2
−∞

L L  16 L  L 32 
= + +  2  cos(∆ωt ) = 1 + 2 cos(∆ωt ) 
4 4  9π  2  9π 
where I used
+∞ L
2
∫ xψ 1 ( x)ψ 2 ( x)dx =
−∞
L ∫0
x sin(πx / L) sin(2πx / L)dx

L
1
x(cos(3πx / L) − cos(πx / L) )dx
L ∫0
=

L L
1 1 2 L  1  16 L
=
L0∫ x cos(3πx / L)dx − ∫ x cos(πx / L)dx = 2 1 −  = 2
L0 π  9  9π
and
L 2 3π
1 1 L  L 3π − 2L
L0∫ x cos(3πx / L)dx =  
L  3π  ∫ y cos( y)dy = (3π )
0
2
(cos y + y sin y ) 0 =
(3π ) 2
,

L 2π
1 1L L π − 2L
L0∫ x cos(πx / L)dx =   ∫ y cos( y )dy = 2 (cos y + y sin y ) 0 = 2 .
L π  0 π π
The easiest way to calculate <px> is as follows:
d < x > mL d  32  16mL
< p x >= m = 1 + 2 cos(∆ωt )  = − ∆ω sin( ∆ωt )
dt 2 dt  9π  9π 2
16mL 3π 2h 8h
=− sin( ∆ωt ) = − sin( ∆ωt )
9π 2mL
2 2
3L
The hard way is as follows

Exam 2 Solutions Page 6 of 7 March 9, 2006


PHY3063 Spring 2006 R. D. Field

+∞ +∞
d
< p x >= ∫ Ψ ( x, t )( px )op Ψ ( x, t )dx = −ih ∫ Ψ ∗ ( x, t )

Ψ ( x, t )dx
−∞ −∞
dx
+∞
 1 1 ∗ d  1 1 
= −ih ∫  ψ 1∗ ( x)e + iω1t + ψ 2 ( x)e + iω 2 t   ψ 1 ( x)e − iω1t + ψ 2 ( x)e − iω 2 t dx
−∞  2 2  dx  2 2 
+∞ +∞
− ih dψ ( x ) − ih dψ ( x )
=
2 −∞∫ ψ 1∗ ( x) 1 dx +
dx 2 −∞∫ ψ 2∗ ( x) 2 dx
dx
+∞ +∞
− ih + iω1t − iω 2 t dψ ( x ) ih dψ ( x )
=
2
e e ∫
−∞
ψ 1∗ ( x) 2 dx − e + iω 2 t e − iω1t ∫ ψ 2∗ ( x) 1 dx
dx 2 −∞
dx
ih + iω1t − iω 2 t  8  ih + iω 2 t − iω1t  8 
= 12 < p x >1 + 12 < p x > 2 − e e − − e e  
2  3L  2  3L 
=
3L
e (
ih 4 + i∆ωt
− e − i∆ωt ) 8h
= − sin( ∆ωt )
3L
where I used
+∞
dψ 2 ( x ) 2 2π
L

∫ ψ 1 ( x) dx dx = L L ∫0 sin(πx / L) cos(2πx / L)dx


−∞

1 2π
L
=
L L ∫ (sin(3πx / L) − sin(πx / L))dx
0

1 2π 1 2π
L L
4 1  8
=
L L 0∫ sin(3πx / L)dx −
L L 0∫ sin(πx / L)dx =  − 1 = −
L3  3L
and
L 3π
1 1 L 1 3π 2
L0∫ sin(3πx / L)dx = ∫
L 3π 0
sin( y )dy = − (cos y ) 0 =
3π 3π
L π
1 1L 1 π 2
L0∫ sin(πx / L)dx = ∫
Lπ 0
sin( y )dy = − (cos y ) 0 =
π π
and
+∞
dψ 1 ( x ) 2π
L

∫ ψ 2 ( x) L L ∫0
sin( 2πx / L) cos(πx / L)dx

dx =
−∞
dx

L
(sin(3πx / L) + sin(πx / L) )dx
L L ∫0
=

1π 1π
L L
2 1  8
=
LL0∫ sin(3πx / L)dx +
LL0∫ sin(πx / L)dx =  + 1 =
L  3  3L

Exam 2 Solutions Page 7 of 7 March 9, 2006

Вам также может понравиться